LSAT and Law School Admissions Forum

Get expert LSAT preparation and law school admissions advice from PowerScore Test Preparation.

 Administrator
PowerScore Staff
  • PowerScore Staff
  • Posts: 8917
  • Joined: Feb 02, 2011
|
#40809
Complete Question Explanation
(The complete setup for this game can be found here: lsat/viewtopic.php?t=13302)

The correct answer choice is (D)

If boxes 2, 3, and 4 all contain balls of the same color, then those balls must be green or red (from the distributions, there are never more than two white balls, and thus the three boxes could never contain three white balls). Here are both solutions:

Boxes 2, 3, and 4 contain green balls

When these three boxes contain green balls, then the 3-2-1 distribution is in effect. Thus, two red balls and one white ball remain to be placed. In order to satisfy the second rule, the two red balls must be in boxes 5 and 6, and in order to satisfy the third rule, the white ball must be in box 1, leading to the following solution:

PT63_Game_#4_#23_diagram 1.png
Boxes 2, 3, and 4 contain red balls

When these three boxes contain red balls, initially it appears as though either the 2-3-1 or 1-4-1 distribution could be in effect. However, in order to satisfy the second rule, a green ball must be in box 1, and in order to satisfy the third rule, a WG block must be in boxes 5 and 6, leading to a 2-3-1 distribution and the following solution:

PT63_Game_#4_#23_diagram 2.png
Thus, when boxes 2, 3, and 4 contain balls of the same color, either the 3-2-1 distribution or the 2-3-1 distribution is in effect. In each scenario there is only one white ball, and thus answer choice (D) is correct.
You do not have the required permissions to view the files attached to this post.

Get the most out of your LSAT Prep Plus subscription.

Analyze and track your performance with our Testing and Analytics Package.